Search found 15 matches

by petrifiedbutstanding

Tue Aug 20, 2013 9:30 pm
Forum: Sentence Correction
Topic: We who
Replies: 22
Views: 13072

Is this form correct?

Hi.. I'm prepping for SC at the moment and read this in a leading business journal, and want to know whether such a representation is correct: "The board did debate on it and were not pleased with it". I would have thought that it's wrong and that you could much rather say: "The board...

by petrifiedbutstanding

Tue Jun 04, 2013 10:19 pm
Forum: Sentence Correction
Topic: Is this form correct?
Replies: 0
Views: 821

Am I glad I got that.. Especially because I solved it mentally..!

by petrifiedbutstanding

Tue Jun 04, 2013 11:17 am
Forum: Data Sufficiency
Topic: absolute value
Replies: 37
Views: 21569

22. Neanderthals had a vocal tract that resembled those of the apes and so were probably without language, a shortcoming that may explain why they were supplanted by our own species. (A) Neanderthals had a vocal tract that resembled those of the apes Those is incorrect. Subject-verb error since voc...

by petrifiedbutstanding

Tue Jun 04, 2013 8:13 am
Forum: Sentence Correction
Topic: Neanderthals had a vocal tract
Replies: 75
Views: 43153

(A) Annual safety inspections ensure that car tires are replaced before they grow old. This further adds to the intrigue as to why there should be accidents at all. (B) Drivers often become overconfident after their cars have passed a thorough safety inspection. This cannot explain a surge in accide...

by petrifiedbutstanding

Tue Jun 04, 2013 8:04 am
Forum: Critical Reasoning
Topic: Car Safety Requirement
Replies: 43
Views: 25396

GMATGuruNY wrote: A negative fraction won't work in statement 1 because a negative fraction raised to an odd power gets bigger: (-1/2)^3 = -(1/8), and -1/8 > - 1/2.

So to satisfy both statements, 0 < a < 1. Sufficient.

The correct answer is C.
I completely missed that! I don't believe it..

by petrifiedbutstanding

Wed May 29, 2013 12:08 am
Forum: Data Sufficiency
Topic: Inequality!!
Replies: 55
Views: 28560

I see you've already answered that. So is this a rule of thumb when you notice "not necessarily" on the GMAT?

by petrifiedbutstanding

Tue May 28, 2013 11:56 pm
Forum: Problem Solving
Topic: Counting - Six mobsters
Replies: 59
Views: 27905

I recently came across this question, and I quite like it. Six mobsters have arrived at the theater for the premiere of the film “Goodbuddies.� One of the mobsters, Frankie, is an informer, and he's afraid that another member of his crew, Joey, is on to him. Frankie, wanting to keep Joey in his...

by petrifiedbutstanding

Tue May 28, 2013 11:52 pm
Forum: Problem Solving
Topic: Counting - Six mobsters
Replies: 59
Views: 27905

If a jury of 12 people is to be selected randomly from a pool of 15 potential jurors, and the jury pool consists of 2/3 men and 1/3 women, what is the probability that the jury will comprise at least 2/3 men? a. 24/91 b. 5/91 c. 2/3 d. 67/91 e. 84/91 Very similiar to the last problem that I posted ...

by petrifiedbutstanding

Tue May 28, 2013 11:30 pm
Forum: Problem Solving
Topic: I Suck at Probability need desperate help
Replies: 69
Views: 58638

The answer is C. I made the decision plainly based on the fact that "interact among" sounded wrong.

Secondly, "interacted with themselves" implies that each member of the group interacted with himself or herself. And that's ridiculous.

by petrifiedbutstanding

Fri May 24, 2013 10:08 pm
Forum: Sentence Correction
Topic: Each other Vs One Other
Replies: 50
Views: 31559

I said A. Here were my reasons: B >> Use of advanced technology has nothing to do with the conclusion. The conclusion is more concerned with continually generating wealth for environmental programs. C >> Again. No relation to the conclusion. D >> The passage says nothing of the sort. It goes contrar...

by petrifiedbutstanding

Mon May 20, 2013 8:37 am
Forum: Critical Reasoning
Topic: LSAT--Columnist
Replies: 46
Views: 23473

I got to C, but used a different technique; testing numbers. Is X>K? = Y/N question (1) 2^x.2^k = 4 Possible value combinations for x & k that total to four include: x = 1, k = 1. Here x = k x = 0, k = 2. Here x < k x = 2, k = 0 . Here x > k As there are multiple possibilities for X>K, we can e...

by petrifiedbutstanding

Mon May 20, 2013 8:24 am
Forum: Data Sufficiency
Topic: Is x > k?
Replies: 36
Views: 26318

I went with D because: A >> This was very limited in scope. It was only restricted to para 2. B >> Again restricted. Was only discussed in 1 para. C >> I thought this was rather disconnected from the passage. E >> I didn't see any reference to coelenterates. D >> This was the best choice that summed...

by petrifiedbutstanding

Mon May 20, 2013 7:55 am
Forum: Reading Comprehension
Topic: Clams
Replies: 56
Views: 33318

IMO is E.

This is my explanation:
A. doesn't make any sense.
B. vague.
C. wrong tense. Second half of the sentence is acceptable, not the first.
D. wrong tense. Second half of the sentence is acceptable, not the first.
E. Grammatically correct.

by petrifiedbutstanding

Thu May 09, 2013 11:35 pm
Forum: Sentence Correction
Topic: Election time
Replies: 91
Views: 41735

In a certain game, a large bag is filled with blue, green, purple and red chips worth 1, 5, x and 11 points each, respectively. The purple chips are worth more than the green chips, but less than the red chips. A certain number of chips are then selected from the bag. If the product of the point va...

by petrifiedbutstanding

Thu May 09, 2013 11:24 pm
Forum: Problem Solving
Topic: More Primes
Replies: 93
Views: 45570